Đến nội dung

Tuan Hoang Nhat

Tuan Hoang Nhat

Đăng ký: 06-04-2015
Offline Đăng nhập: 02-07-2015 - 22:54
***--

#569550 Bánh canh chém gió về kì thi IMO 2015

Gửi bởi Tuan Hoang Nhat trong 02-07-2015 - 22:49

Trước tiên em xin chúc cho tất cả các anh dự thi IMO bên Thái Lan sẽ đạt kết quả cao nhất. Em được biết anh Nguyễn Huy Hoàng qua chương trình đường lên đỉnh Olympia, em rất ngưỡng mộ anh ấy. Em chúc anh sẽ giành được HCV ạ! 

Em cũng biết 1 anh qua chương trình đường lên đỉnh Olimpia, không biết phải anh này không :D




#569539 $\frac{1}{1+a+ab}+\frac{1}{...

Gửi bởi Tuan Hoang Nhat trong 02-07-2015 - 21:54

1/ Cho a, b, c, d là các số thực dương thỏa mãn abcd=1. Chứng minh $\frac{1}{1+a+ab}+\frac{1}{1+b+bc}+\frac{1}{1+c+cd}+\frac{1}{1+d+da}>1$

2/ Cho a, b, c là các số thực dương. Chứng minh $1<\frac{a^{2}}{a^{2}+bc}+\frac{b^{2}}{b^{2}+ca}+\frac{c^{2}}{c^{2}+ab}<2$

 

Giải bằng phương pháp làm trội càng tốt ạ.

Câu 1:

Do $abcd=1$ nên có thể đặt: $a=\frac{m}{n};b=\frac{n}{p};c=\frac{p}{q};d=\frac{q}{m}$

Khi đó BĐT trở thành:$\sum \frac{m}{m+n+p}>1$ 

Hiển nhiên đúng vì $\sum \frac{m}{m+n+p}>\sum \frac{m}{m+n+p+q}=1$ 




#569472 $\sum (\frac{a}{a+b})^2+3\geq \f...

Gửi bởi Tuan Hoang Nhat trong 02-07-2015 - 16:02

Ngoài lề: Các anh cho em hỏi hạn nộp bài là như thế nào ạ :D




#569414 $\sum (\frac{a}{a+b})^2+3\geq \f...

Gửi bởi Tuan Hoang Nhat trong 02-07-2015 - 10:34

 Bài toán: Cho các số thực dương $a,b,c$. CMR:

 

    $(\frac{a}{a+b})^2+(\frac{b}{b+c})^2+(\frac{c}{c+a})^2+3\geq \frac{5}{2}(\frac{a}{a+b}+\frac{b}{b+c}+\frac{c}{c+a})$

 

 

 

 

 

P/s: Đây là bài toán nằm trên báo THTT ,do đã hết thời lượng gửi bài nên up lên đây mọi người thảo luân

Câu này em biến đổi tương đương rồi sử dụng BĐT: $x^3+y^3\geq xy(x+y)$ :(




#569384 CMR: $\sum \frac{a^{3}}{b^{2...

Gửi bởi Tuan Hoang Nhat trong 01-07-2015 - 23:22

Chứng minh BĐT: 

$\frac{a^{3}}{b^{2}-bc+c^{2}}+\frac{b^{3}}{c^{2}-ca+a^{2}}+\frac{c^{3}}{a^{2}-ab+b^{2}}\geq \frac{3(ab+bc+ca)}{a+b+c}$  trong đó a, b, c là các số dương 

BĐT này tương đương:

$\sum \frac{a^3}{b^3+c^3}(b+c)\geq \frac{3\sum ab}{\sum a}$

Ta có:

$\sum \frac{a^3}{b^3+c^3}(b+c)=\sum \frac{a^3+b^3+c^3}{b^3+c^3}(b+c)-2(a+b+c)=\frac{1}{2}\sum (b^3+c^3)\sum \frac{b+c}{b^3+c^3}$

$\geq \frac{1}{2}(\sum \sqrt{a+b})^2-2(a+b+c)=\sum \sqrt{(a+b)(a+c)}-(a+b+c)$

Cần chứng minh:$\sum \sqrt{(a+b)(a+c)}-(a+b+c)\geq \frac{3\sum ab}{\sum a}$

Đặt: $2x^2=b+c;2y^2=c+a;2z^2=a+b$

Khi đó BĐT sẽ trở thành:

$\sum x^4+\sum xy(x^2+y^2)+\sum x^2yz\geq 4\sum x^2y^2$

Theo BĐT Schur bậc 4 thì:$\sum x^4+\sum x^2yz\geq \sum xy(x^2+y^2)$

Đồng thời cũng áp dụng BĐT AM-GM thì:

$\sum xy(x^2+y^2)\geq 2\sum x^2y^2$

BĐT được chứng minh :(




#569191 Tìm nghiệm nguyên: $3^{x}+4^{y}=7^{z}$

Gửi bởi Tuan Hoang Nhat trong 30-06-2015 - 23:08

Trường hợp: $y \geq 2$

Ta có: $4^y=7^z-3^x\equiv (-1)^z-(-1)^x\equiv 0$ (mod 4)

Do đó x và z cùng tính chẵn lẻ.

Nếu cả x và z cùng lẻ thì $7^z \equiv 7$ (mod 8)  và $3^x \equiv 3$ (mod 8)

Do đó: $7^z-3^x \equiv 4$ (mod 8)

Dễ dàng giải tiếp.

Nếu cả x và z cùng chẵn. Khi đó đặt $x=2b$ và $z=2a$

Đến đây giải như thế nào :(




#569186 Tìm nghiệm nguyên: $3^{x}+4^{y}=7^{z}$

Gửi bởi Tuan Hoang Nhat trong 30-06-2015 - 22:24

Tìm nghiệm nguyên của phương trình:

$3^{x}+4^{y}=7^{z}$

Giờ sẽ xét với cả 3 số x,y,z đều $\geq 0$ thực ra cũng phải xét thêm một vài trường hợp âm dương nữa nhưng mà cứ tạm bỏ qua :(

Trường hợp 1: $y=1$

Khi đó thì phương trình trở thành: $3^x+4=7^z$

Với $x \geq 3$ thì $7^z\equiv 4$ (mod 27)

Khi đó thì $z \equiv 8$ (mod 9) 

Đặt: $z=9k+8$

Khi đó $3^x+4=7^{9k+8}\equiv 16$ (mod 37)

=> $3^x \equiv 12$ (mod 37)  (1)

Nhận thấy: $3^x+4$ chia hết cho 7

Do đó: $x \equiv 1$ (mod 6)

=> $x \equiv 1,7,13$ (mod 18)

=> $3^x \equiv 3,4,30$ (mod 37) (2)

(1) và (2) mâu thuẫn nhau, do đó $x<3$. Từ đó nhận thấy với $y=1$ thì $x=z=1$, trường hợp $x=2$ loại 

Spoiler




#569184 Tìm nghiệm nguyên: $3^{x}+4^{y}=7^{z}$

Gửi bởi Tuan Hoang Nhat trong 30-06-2015 - 22:11

Tìm nghiệm nguyên của phương trình:

$3^{x}+4^{y}=7^{z}$

Trường hợp 1:cả x,y,z đều $\leq 0$

khi đó cho x=-a;y=-b;z=-c thì $a,b,c \geq 0$

Phương trình này sẽ tương đương với:

$\frac{1}{3^a}+\frac{1}{4^b}=\frac{1}{7^c}<=>\frac{3^a4^b}{3^a+4^b}=7^c$

=> $3^a4^b$ chia hết cho $7^c$

Bắt buộc $c=0$

Đến đây có thể giải quyết dễ dàng :(

Spoiler




#569004 $\sqrt{(a^2b+b^2c+c^2a)(ab^2+bc^2+ac^2)}\geq abc+...

Gửi bởi Tuan Hoang Nhat trong 29-06-2015 - 23:15

Chứng minh với mọi a, b, c dương:

$\sqrt{(a^2b+b^2c+c^2a)(ab^2+bc^2+ac^2)}\geq abc+\sqrt[3]{abc(a^2+bc)(b^2+ac)(c^2+ab)}$

Chia cả hai vế của BĐT cho abc rồi đặt: $x=\frac{a}{b};y=\frac{b}{c};z=\frac{c}{a}$ thì $xyz=1$

BĐT trở thành:

$\sqrt{(x+y+z)(xy+yz+xz)}\geq 1+\sqrt[3]{(\frac{x}{z}+1)(\frac{y}{x}+1)(\frac{z}{y}+1)}$

Từ $xyz=1$ ta thu được:

$(\sum x)(\sum xy)=\prod (x+y)+xyz=\prod (x+y)+1$

Khi đó đặt: $\sqrt[3]{\prod (x+y)}=m$ thì BĐT trở thành: $\sqrt{m^3+1}\geq 1+m$

Áp dụng AM-GM dễ thấy $m\geq 2$

Từ đó biến đổi tương đương => ĐPCM




#568995 Chứng minh $(a+bc)^{2}+(b+ca)^{2}$$+(c+ab)...

Gửi bởi Tuan Hoang Nhat trong 29-06-2015 - 21:55

Cho a,b,c là các số không âm.Chứng minh rằng:

$(a+bc)^{2}+(b+ca)^{2}$$+(c+ab)^{2}$$\geq \sqrt{2}(a+b)(b+c)(c+a)$

Câu này ở trên báo THTT số 452, đã được giải trong báo số 456 rồi nhé bạn :(




#568792 Chứng minh rằng $\frac{a^2}{a^2+(b+c)^2}+\...

Gửi bởi Tuan Hoang Nhat trong 28-06-2015 - 23:02

Bạn có tài liệu nào về cách chọn hệ số k như của bạn không? Bạn làm nhưng có nhiều chỗ mình không biết ở đâu ra như Vì sao xét $\frac{21}{8}$,...

Bạn thử lên google seach: Phương pháp hệ số bất định UCT thử xem. :D

Mình nhác tìm quá :(




#568786 Chứng minh rằng $\frac{a^2}{a^2+(b+c)^2}+\...

Gửi bởi Tuan Hoang Nhat trong 28-06-2015 - 22:43

Bạn có thể giải chi tiết được không? Mình không hiểu cách các bạn tìm số k cho lắm.

Như cách của bạn Long được rồi, còn cách tớ (suy nghĩ theo hướng chủ quan) là như thế này :D

Sau khi chuẩn hóa thì BĐT trở thành:

$\sum \frac{a^2}{2a^2-6a+9}\geq \frac{3}{5}$

Với $a,b,c\leq \frac{21}{8}$ thì $\sum \frac{a^2}{2a^2-6a+9}\geq \sum \frac{12a-7}{25}<=>\sum (a-1)^2(8a-21)\leq 0$

Từ đó dẫn đến ĐPCM

Nếu trong 3 số a,b,c có một số $\geq \frac{21}{8}$, giả sử đó là số a, vì $a+b+c=3$ nên $b,c\leq 3-\frac{21}{8}=\frac{3}{8}$

Và có $a\leq 3$ nên $\frac{a^2}{a^2+(b+c)^2}>\frac{21^2}{8^2}:(3^2+4.\frac{3^2}{8^2})=\frac{49}{68}>\frac{3}{5}$

Không liên quan




#568781 Chứng minh rằng $\frac{a^2}{a^2+(b+c)^2}+\...

Gửi bởi Tuan Hoang Nhat trong 28-06-2015 - 22:20

Ta cần tìm hệ số k sao cho $\frac{a^{2}}{2a^{2}-6a+9}\geq \frac{1}{5}+k(a-1)$ Ta có $-\frac{5}{2a^{2}-6a+9}+1=\frac{-3(a-1)(a+3)}{2a^{2}-6a+9}$ Khi a=1 thì $\frac{a+3}{2a^{2}-6a+9}=\frac{4}{5}$ Vậy ta cần chứng minh $\frac{(a-1)(a+3)}{2a^{2}-6a+9}\leq \frac{4}{5}(a-1)$ BĐt này $\frac{(a-1)(a+3)}{2a^{2}-6a+9}\leq \frac{4}{5}(a-1)$ mình chứng minh không được :(

Hệ số $k= \frac{12}{25} $ mà bạn :D

Và khi đó xét 2 trường hợp nữa là ra




#568778 $\frac{a^5-a^2}{a^5+b^2+c^2}+\frac{b^...

Gửi bởi Tuan Hoang Nhat trong 28-06-2015 - 22:06

Cho $a,b,c$ là các số thực dương thỏa mãn $a^2+b^2+c^2\geq 3$. Chứng minh rằng $\frac{a^5-a^2}{a^5+b^2+c^2}+\frac{b^5-b^2}{b^5+a^2+c^2}+\frac{c^5-c^2}{c^5+b^2+c^2}\geq 0$

Hình như giả thiết phải là $abc \geq 1$ chứ nhỉ, nếu vậy thì bạn có thể xem ở đây:

http://diendantoanho...hi-mo-các-nước/




#568776 Chứng minh rằng $\frac{a^2}{a^2+(b+c)^2}+\...

Gửi bởi Tuan Hoang Nhat trong 28-06-2015 - 21:59

mình chưa hiểu chỗ này, đề đâu có cho đâu bạn  :(

Vì bất đẳng thức đồng bậc nên có thể chuẩn hóa như vậy đó bạn

 

Sao mình vẫn không ra nhỉ :( Mình chưa học tiếp tuyến nhưng mà mình chuẩn hóa $a+b+c=3$ rồi làm theo kiểu tìm hệ số k thì không chứng minh được :(

Làm theo hệ số k cũng là tiếp tuyến đó bạn :D bạn thử làm lại xem